1. x/5 + 10 = 40
2. 4x - 36 = -2
i would really appreciate it if you'd help me

Answers

Answer 1

Answer:

1.) x = 150

2.) x = 8.5

Step-by-step explanation:

1.)

[tex]\frac{x}{5}+10=40\\\\\frac{x}{5}=30\\\\x=150[/tex]

2.)

[tex]4x-36=-2\\\\4x=34\\\\x=8.5[/tex]

Answer 2

Answer:

1. x = 150

2. x = 8.5

Step-by-step explanation:

1. x/5 + 10 = 40

Get x by itself and then solve for x.

First, subtract 10 from both sides

x/5 + 10 - 10 = 40 - 10

x/5 = 30

Then, multiply by 5 to get x by itself.

x/5 = 30

x/5 * 5 = 30 * 5

x = 150

2. 4x - 36 = -2

Get x by itself and then solve for x.

First, add 36 to both sides

4x - 36 + 36 = -2 + 36

4x = 34

Then, Divide each side by 4

4x/4 = 34/4

x = 8.5


Related Questions

If 2 – x > x – 2 then the solution of x is

Answers

[tex]\\ \sf\longmapsto 2-x\gt x-2[/tex]

[tex]\\ \sf\longmapsto 2+2\gt x+x[/tex]

[tex]\\ \sf\longmapsto 4\gt 2x[/tex]

[tex]\\ \sf\longmapsto x\lt \dfrac{4}{2}[/tex]

[tex]\\ \sf\longmapsto x\lt 2[/tex]

Add x + 2 to both sides
2 + 2 > x + x
2x < 4
x < 2

What is the domain in roster notation(numerical order) x= -2,-1,1,3,3 y=3,-3,0-3,3

Answers

Answer:

Domain is {x|x = -2, -1, 1, 3}

Step-by-step explanation:

I. When domain is x.value and renge is y.value

Got more question or something to discuss? leave comment below

PLS HELP WILL MARK AS BRAINLIEST ​

Answers

Answer:

SEE THE IMAGE FOR SOLUTION .

HAVE A GREAT DAY

Answer:

11. D

12. C

13. D

14. B

15. E

Step-by-step explanation:

11.

1/3 + 1/2 = 2/6 + 3/6 = 5/6

A is 3/4

B is 2/3

C is 9/10

D is 4/5

E is 2/5

the closest to 5/6 is 4/5.

12.

this is totally easy. just simply additions and one subtraction (to see what is left).

8+10+9 = 27

so, he has

60 - 27 = 33 hours left

13.

the mean value of a series of equal weighted numbers is simply their sum divided by the amount of numbers.

so,

(2+5+3+6+4)/5 = 20/5 = 4

14.

a scale factor is always applied to all dimensions, otherwise the result will be distorted. and that would be especially bad for a map.

so, clearly the width of the larger map is 3 times the width of the smaller map. therefore, the length of the larger map has to be also 3 fine the length of the smaller map.

3×6 = 18

15.

3t - 10 = 8

3t = 18

t = 6

Can someone tell me the asnwer to this ASAP?

Answers

Answer:

[tex]2\sqrt{2} \\2[/tex]

Step-by-step explanation:

[tex]\sqrt{8} = \sqrt{4*2} = \sqrt{4} * \sqrt{2} = 2*\sqrt{2} = 2\sqrt{2} \\\\\sqrt[3]{8} = \sqrt{2*2*2} = 2[/tex]

Answer:

I noticed that  [tex]\sqrt{8\\}[/tex] does not have 3 on it like the second one. And i wonder that the answers would be different from each other

[tex]2\sqrt{2}[/tex]

2

Translate the sentence into an inequality.
Eight subtracted from b is less than or equal to 27.



THIS IS MY LASY!! 20Points

Answers

Step-by-step explanation:

Note that less means (-) and less than or equal means ≤

b - 8 ≤ 27

Hey there!

The answer is:

[tex]b - 8 \leqslant 27[/tex]

Hope it help you

can any one solve this problem ​

Answers

Answer:

sorry i can't

please mark me as brainlest

Find the sum of
2x^2- 8x– 6 and 9x^2-8

Answers

Answer:

11x^2-8x-14

Step-by-step explanation:

(2x^2- 8x– 6)+(9x^2-8) = 11x^2-8x-14

Answer:

11x^2-8x-14

Step-by-step explanation:

2x^2- 8x– 6 and 9x^2-8

solution:

2x^2- 8x– 6 +(9x^2-8)

=2x^2-8x–6+9x^2-8

=2x^2+9x^2-8x-6-8

=11x^2-8x-14

Solve 29 – 3:= 5x + 5 for x.​

Answers

Answer:

x=4.2

Step-by-step explanation:

29-3=5x+5

21=5x, x=21/5=4.2

X=4.2 because 29-3=5x +5 for d

g(x) = f(x) +2
if f(x) is 2x

Answers

Answer:

You didn't put the question well but if I am right, the question is asking of g(x) if f(x) is 2x.

then g(x)=2x+2

Translate to an algebraic expression.
29 more than n

Answers

Answer: n+29
20 letters…

93. Let f(x) = 3x - 1 and g(x) = x + 5. Evaluate the following limits. (a) lim x→3 f(g(x)]
(b) lim x→3 g[f(x)]

Answers

Answer:

a) 23 and b) 13

Step-by-step explanation:

I. If f(g(x)) is composit function

Find fog and gof

     fog = 3(g) - 1

           = 3(x+5) -1

           = 3x + 15 -1

           = 3x + 14

    gof = (f) + 5

          = (3x - 1) +5

          = 3x + 4

II. Fill in a) and b) question

            lim x -> 3 fog

           = 3(x) + 14

           = 3(3) + 14

          = 9 + 14

       a)  = 23

            lim x-> 3 gof

           = 3x + 4

           = 3(3) + 4

           = 9 + 4

        b)  = 13

Got more question or something to discuss, leave comment below

Pls help me I’m not very smart

Answers

Answer:

39.90

Step-by-step explanation:

multiply 95x42, ur not smart if you think u arent.

39.90 is the answer keep up the work

The price of a printer is $349.00, plus 6% sales tax. What is the sales tax on this printer in dollars and cents?​

Answers

$349.00 x 0.06 (6%) = $20.94 sales tax

The sales tax on this printer is $20.94

What is sales tax?

A sales tax is a consumption tax imposed by the government on the sale of goods and services.

Given that, the price of a printer is $349.00 having a 6% sales tax.

Sales tax = 6% of $349.00

$349.00 x 0.06 = $20.94

Hence, the sales tax on this printer is $20.94

For more references on sales tax, click;

https://brainly.com/question/29442509

#SPJ6

Please help me I’m struggling! I’ll give a brainliest!!
Collinear points lie on the same line. Find the value of k for which the following points are collinear: (k,3), (-3,2), (-1,1)

Answers

Answer:

k=-5

Step-by-step explanation:

For the points to be collinear any two pair must have the same slope

We find the slope of the 2nd and 3rd points

=1-2/-1--3

=-1/2

Now finding the slope of the 1st and any other point say point 3

1-3/-1-k=-½

-2/-1-k=-½

-4=-1(-1-k)

-4=1+k

k=-4-1

k=-5

PROOF

if the first point is (-5,3)

slope of point 1 and 2

=2-3/-3--5

=-1/2

3/4x + 4/7y = 5/4x -1...solve for y​

Answers

Answer:

[tex] \frac{3}{4} x + \frac{4}{7} y = \frac{5}{4} x - 1 \\ \frac{4}{7} y = \frac{5}{4} x - 1 - \frac{3}{4} x \\ \\ \frac{4}{7} y = \frac{5x - 4 - 3x}{4} \\ \frac{4}{7} y = \frac{2x - 4}{4} \\ \\ y = \frac{2x - 4}{4} \div \frac{4}{7} \\ y = \frac{2x - 4}{4} \times \frac{7}{4} \\ \\ y = \frac{14x - 28}{16} \\ \\ y = \frac{14x}{16} - \frac{28}{16} [/tex]

I hope I helped you^_^

Can somebody please help? I been struggling to answer this!

Answers

Answer:

the first option is the answer. I am sure of this

Answer:

the first option is the answer

evaluate the expression (-2) - ( -5)

Answers

Answer:

The answer is 3

Step-by-step explanation:

(-2)+5=3

Tìm tập giá trị y = x² - căn 4 - x²

Answers

Answer:

0 and 2 s the answer

someone help please :)

Answers

Answer:

its basically a Cross

the point they cross is p

and the 2 lines are AB and RS

I hope I'm correct.And yea

.hope it helps

1. Identify the terms in the expression
1.3x - 2.7x2
-
5.4x + 3


50 points whoever aswners it

Answers

Answer:

- and+

Step-by-step explanation:

Answer:

1.3x, -2.7, x^2 (x squared), -5.4x, +3 are the terms. hope this helps

Step-by-step explanation:

- Zombie

Simplify each expression completely.
7[3+2(x-4)-6(x-4)]

Answers

Answer:

21 + 14x - 56 - 42x + 168

= 133  - 28x

Step-by-step explanation:

Answer:

19 - 4x

Step-by-step explanation:

[tex]3 + 2(x - 4) - 6(x - 4)[/tex]

[tex]3 + 2x - 8 - 6(x - 4)[/tex]

[tex]3 + 2x - 8 - 6x + 24[/tex]

[tex]19 + 2x - 6x[/tex]

[tex]19 - 4x[/tex]

How many solutions are there to the equation below?
145 = 10x - 8x
O A. infinitely many
O B. 2
O c. 1

Answers

Answer:

c. one solution

Step-by-step explanation:

[tex]145 = 10x - 8x \\ 2x = 145 \\ x = 72.5[/tex]

Answer:

C

Step-by-step explanation:

yes

this is the answer for sure

4 The equation 5W + 3 = 4w + 9 is modeled below. w W 1 W W 1 1 W 1 1 W W 1 1 1 What value of w makes this equation true? Record your answer and fill in the bubbles on your answer document. Be sure to use the correct place value. MI - SI – PSA- S-​

Answers

Answer:

w = 6.

Step-by-step explanation:

5w + 3 = 4w + 9

5w - 4w = 9 - 3

w = 6

Then, check your work!

5(6) + 3 = 4(6) + 9

30 + 3 = 24 + 9

33 = 33

So, w = 6 will be the correct answer.

Hope this helps!

Helppp pleaseeeee

Which expression is equivalent to

Answers

Answer:

The second option

Step-by-step explanation:

First, simplify the bracket

[tex] \frac{4r {}^{3} s {}^{2} t {}^{4} }{2r {}^{4} st {}^{6} } [/tex]

Apply Quotient of Powers

[tex]2 {r}^{ - 1} s {t}^{ - 2} [/tex]

Which equals

[tex] \frac{2s}{rt {}^{2} } [/tex]

Raise this to the fifth power

[tex] \frac{32s {}^{5} }{ {r}^{5} t {}^{10} } [/tex]

Multiply by the outsides numbers(5^r^6 and t^4

[tex] \frac{160 {}^{} r {}^{6}s {}^{5} t {}^{4} }{r {}^{5} t {}^{10} } [/tex]

[tex]160r {}^{5} t {}^{ - 6} [/tex]

[tex] \frac{160r {}^{}s {}^{5} }{t {}^{6} } [/tex]

Find the difference between temperatures of 17 degrees celcius above zero and 12 degrees celcius below zero

Answers

Answer:

29° C

Step-by-step explanation:

-17(-12)

=29

Hope this helps

Answer:

29C

Step-by-step explanation:

Above zero temperature = +17C

Below zero temperature = -12C

Difference= 17 - (-12C)

                 = 17 + 12 C

                 =29C

e) If the radede circle is 7em, then find the area of circle. al- (SSS. x at Solna -​

Answers

Hi

area of circle, which is call in proper math terme circumference ( latin for going around) is given by this formula :

2 times radius x pi

As math is all about knowing your formula and training, I let you have a shot. You wont progress other wise.

Ask in comment section if you are still blocked.

6) find HJ
8)find IK

Answers

Answer:

HJ = 17

IK = 30

Step-by-step explanation:

to find HI :

GH : 2

HI : ?

IJ : 12

GI : 7

HI = GI - GH

HI = 7- 2

HI = 5

HI + IJ = HJ

5 + 12 = HJ

17 = HJ

To find IK

IJ = ?

JK = 12

KL = ?

IL = 49

JL = 31

To find IK we need to find KL and IJ

KL = JL - JK

KL = 31 - 12

KL = 19

IJ = IL - (JK + KL)

IJ = 49 - 31

IJ = 18

IK = IJ + JK

IK = 18 + 12

IK = 30

A clothing company buys t-shirts for $10 and sells them for $100. What is the percentage mark up on
t-shirts? Make sure you show at least one step of working

Answers

Answer:

900%

Step-by-step explanation:

Starting price : 10

End price: 100

100-10 = 90

90/10 = 9 = 900%

If my answer is incorrect, pls correct me!

If you like my answer and explanation, mark me as brainliest!

-Chetan K

Using the numbers 2,7,13,11,5 can you get the total of 23? (You must use all numbers, no negatives, no using each number more than once, and only use Subtraction, Multiplication, addition, and division, you can also group numbers)

Answers

Answer:

(2-7)/5 + 13 + 11

Step-by-step explanation:

There's a lot of guess and check that needs to go into this to make this work, but here's what I got:

(2-7)/5 + 13 + 11

-5/5 + 13 + 11

-1 + 13 + 11

24 - 1

23

HELP PLEASEEEEEEEEEEE

Answers

Answer:

The answer you have selected is correct.

Step-by-step explanation:

Answer:

X^2 - 7xy + y^2 +2y

Step-by-step explanation:

For this, they are just trying to test if u know how to expand the brackets, especially for the one on the right, since that is a negative infront of the bracket, u would need to multiply negative to all the values in that bracket

Other Questions
solve 4m plus 2n equals 5n for m. What is the answer ? capital of india???????????? Solve for W.4w=5.12 in the fifth declaration what does petition the king mean? . Evaluate the expression below for x = 4. 6(x+8) (please during the interconversion of state of matter, mass undergoes and change? true or false Answer question worth 25 points plz help! what do modern marriages and Renaissance marriages have in common? At a charity fund-raiser, adult tickets were sold for $10 each and children's tickets were sold for $4 each. Write an algebraic expression for the total amount of money raised from the sale of tickets. How much money was raised if the fundraiser sold 244 adult tickets and 387 children's tickets? A multiple choice test contains 25 questions with 5 answer choices. What is the probability of correctly answering 8 questions if you guess randomly on each question? A car moves with an average speed of 45 mph. How long does a car take to travel 90 miles? Analyzewhether wrestling is fiction or non-fiction. Use article orSource of information that your desiresionany may> If a ribbon that is 30.1 inches long is cut into equal pieces that are 1.75 inches long, how many pieces of ribbon can be created? 1.72 12.7 17.2 172.0 define tense with 2 example. Two major league players got a total of 226 hits. Washington had 18 more hits than Sanchez. Find the number of hits for each player. Identify one constitutional trait of the federal judicial branch or of the Supreme Court designed to ensure the judicial branches independence how do physical systems affect human systems One reason that Paul may have authored Hebrews is that both Hebrews and Philippians mention the humiliation of Christ.a. Trueb. False Lines m and n are parallel lines. If line m has a slope of -1, find the slope of linen. Type a numerical answer in the spaceprovided. If necessary, use the key as a fraction bar. Do not type spaces in your answer.